Okay, I’ll bite on this math argument. The answer is nine. I too came out with 1 but that was my error. The reason for being 9: you have to revisit PEMDAS after each step.

Advertisement

6 / 2 (1 + 2)

6 / 2 (3) <- There is implied multiplication between the 2 and 3. But that means you have an equation that is now only division and multiplication. The brackets are irrelevant. You’ve cancelled them out.

Advertisement

6 / 2 * 3

3 * 3

9

The big thing is, you’ve resolved the brackets when you added 1+2.

Advertisement

And since a trusty TI-83+ calculator agrees, I’m going with it.